--- title: Category $\OO$, Problem Set 4 --- # Humphreys 3.1 Let $\lieg = \liesl(2, \CC)$ and identify $\lambda \in \lieh\dual$ with a scalar. Let $N$ be a 2-dimensional $U(\lieb)\dash$module defined by letting $x$ act as $0$ and $h$ act as $\left(\begin{array}{ll}{\lambda} & {1} \\ {0} & {\lambda}\end{array}\right)$. Show that the induced $U(\lieg)\dash$module structure $M \definedas U(\lieg) \tensor_{U(\lieb)} N$ fits into an exact sequence which fails to split: $$ 0 \to M(\lambda) \to M \to M(\lambda) \to 0 $$ ## Solution [Reference 1](https://math.stackexchange.com/questions/2272891/extension-of-dual-verma-module/2273008#2273008) [Reference 2](https://aip.scitation.org/doi/full/10.1063/1.5121236) > Hence $M\not\in \OO$. We first unpack all definitions in terms of tensor products, using the fact that $M(\lambda) = U(\lieg) \tensor_{U(\lieb)} \CC_\lambda$: \begin{center} \begin{tikzcd} 0 \ar[r] & M(\lambda) \ar[r] & M \ar[r] & M(\lambda) \ar[r] & 0 \\ 0 \ar[r] \ar[u, equal] & U(\lieg) \tensor_{U(\lieb)} \CC_\lambda \ar[u, equal] \ar[r] & U(\lieg) \tensor_{U(\lieb)} N \ar[u, equal] \ar[r] & U(\lieg) \tensor_{U(\lieb)} \CC_\lambda \ar[u, equal] \ar[r] & 0 \ar[u, equal] \\ & 1 \tensor 1 \ar[r, mapsto, "\psi"] & 1 \tensor \vector u \ar[r, mapsto, "\phi"] & 1 \tensor 0 & \\ & & 1 \tensor \vector v \ar[r, mapsto] & 1 \tensor 1 & \end{tikzcd} \end{center} where $N = \spanof_\CC \theset{\vector u, \vector v}$. We make the following claims: 1. The $U(\lieb)$ action defined on $N$ lifts to a $U(\lieg)\dash$action on $M$. 2. This is an exact sequence of $U(\lieg)\dash$modules. 3. $M \not\cong M(\lambda) \oplus M(\lambda)$, showing that this sequence can not split. **Claim 1**: We choose the basis \begin{align*} x = \begin{bmatrix} 0 & 1 \\ 0 & 0 \end{bmatrix} ,\quad h = \begin{bmatrix} 1 & 0 \\ 0 & -1 \end{bmatrix} ,\quad y = \begin{bmatrix} 0 & 0 \\ 1 & 0 \end{bmatrix} \end{align*} and note that in the triangular decomposition $\lieg = \lien^- \oplus \lieh \oplus \lien$, we have \begin{align*} \lien^- &= \CC \cdot x \\ \lieh &= \CC \cdot h \\ \lien^+ &= \CC \cdot y \\ .\end{align*} Since the action is defined over $\lieb = \lieh \oplus \lien$ and $x$ acts by zero, we obtain a $\lieg\dash$action on $N$ which thus extends uniquely to a $U(\lieg)\dash$ action. **Claim 2**: We first note that since the submodule $\CC \cdot \vector u < M$ is closed under the action of $h$ (since $h$ acts by $u\mapsto \lambda u$) and is equal to the image of $\psi$, we can identify $\CC\cdot \vector u \cong \CC_\lambda$ as $U(\lieb)\dash$modules and identify $M(\lambda)$ as a submodule of $N$. Since submodules of $N$ lift to submodules of $\Ind_{\lieb}^\lieg N$, the map $\psi$ is an injection. Moreover, the map $\phi$ is a surjection, since the generator $1\tensor 1$ of $M(\lambda)$ is precisely the image of one of the generators of $M$. To see that the sequence is exact in the middle, we note that by choosing a PBW basis of $\liesl(2, \CC)$ and a basis $\theset{\vector u, \vector v}$ for $N$, we can obtain a basis of $M$ of the form $\theset{y^j \tensor \vector u, y^k \tensor \vector v \suchthat j, k\in \ZZ^{\geq 0}}$. This allows us to identify the lift of the submodule $\CC\cdot \vector u$ to the span of $\theset{y^k \tensor \vector u}$ in $M$. Then $\im \psi \subseteq \ker \phi$ by construction, since $$ \phi(y^k \tensor \vector u) = \phi(y^k(1\tensor \vector u)) = y^k \phi(1\tensor \vector u) = y^k \qty{1 \tensor u} = 0. $$ To see that $\ker \phi \subseteq \im \psi$, we can use the same calculation to explicitly check the map on the remaining basis elements: $$ \phi(y^k \tensor \vector v) = \phi(y^k(1\tensor \vector v)) = y^k \phi(1\tensor \vector v) = y^k \qty{1 \tensor 1} = y^k \tensor 1 \neq 0 .$$ Thus $\ker \phi = \im \psi$, yielding exactness in the middle. **Claim 3**: This follows from the checking the $\lambda\dash$weightspaces of both $M$ and $M(\lambda) \oplus M(\lambda)$. Noting that the matrix $\begin{bmatrix} \lambda & 1 \ 0 & \lambda \end{bmatrix}$ is in Jordan Normal Form, we can read off that the $\lambda$ is an eigenvalue with multiplicity 2, and that the corresponding $\lambda$ eigenspace is 1 dimensional since this is a single Jordan block. However, the $\lambda$ weight space of $M(\lambda) \oplus M(\lambda)$ is of dimension least 2. $\qed$ # Humphreys 3.2 Show that for $M\in \OO$ and $\dim L < \infty$, $$ (M\tensor L)\dual \cong M\dual \tensor L\dual $$ > [Reference for Dual of Sum](https://mathoverflow.net/questions/56255/duals-and-tensor-products) ## Solution We first note that $M\in \OO \implies M = \bigoplus_{\lambda \in \lieh\dual} M_\lambda$ where each $M_\lambda$ is a finite-dimensional weight space. Moreover, $M\dual \definedas \bigoplus_{\lambda \in \lieh\dual} M_\lambda\dual$ is defined to be a direct sum of duals of weight spaces, which are still finite-dimensional. So let $M, N\in \OO$; we will proceed by showing that both $(M\tensor_\CC L)\dual$ and $M\dual \tensor_\CC \dual$ have identical direct sum decompositions. We first have \begin{align*} (M\tensor_\CC L)\dual &\definedas \bigoplus_{\lambda \in \lieh\dual} (M\tensor_\CC L)_\lambda\dual, && \text{the $\lambda$ weight spaces of $M\tensor_\CC L$} \\ &\cong \bigoplus_{\lambda \in \lieh\dual} \qty{\bigoplus_{\alpha+\beta = \lambda} \qty{M_\alpha \tensor_\CC L_\beta } }\dual && \text{by an exercise on the weight spaces of a tensor product} \\ &\cong \bigoplus_{\lambda \in \lieh\dual} \qty{\bigoplus_{\alpha+\beta = \lambda} \qty{M_\alpha \tensor_\CC L_\beta }\dual } && \text{since the inner term is a finite sum}\\ &\cong \bigoplus_{\lambda \in \lieh\dual} \qty{\bigoplus_{\alpha+\beta = \lambda} \qty{M_\alpha\dual \tensor_\CC L_\beta\dual }} &&\text{since the weight spaces are finite-dimensional} ,\end{align*} where we've repeatedly used the fact that $(V\tensor W)\dual \cong V\dual \tensor W\dual$ for finite-dimensional vector spaces, which inductively holds for any finite direct sum of vector spaces. On the other hand, using the fact that \begin{align*} \qty{A\oplus B} \tensor (C\oplus D) &= \qty{ \qty{A\oplus B} \tensor C} \oplus \qty{ \qty{A\oplus B} \tensor D } \\ &= \qty{A\tensor C} \oplus \qty{B\tensor C} \oplus \qty{A\tensor D} \oplus \qty{B\tensor D} \\ \implies \qty{ \bigoplus_{j\in J} A_i} \tensor \qty{ \bigoplus_{k\in K} B_k} &= \bigoplus_{j\in J} \bigoplus_{k\in K} \qty{A_j \tensor B_k} \qtext{by induction} .\end{align*} we can write \begin{align*} M\dual \tensor_\CC L\dual &\definedas \qty{\bigoplus_{\alpha\in \lieh\dual} M_\alpha\dual} \tensor_\CC \qty{ \bigoplus_{\beta \in \lieh\dual} L_\beta\dual } \\ &\cong \bigoplus_{\lambda \in \lieh\dual}\qty{ \bigoplus_{\alpha + \beta = \lambda} \qty{ M_\alpha\dual \tensor_\CC L_\beta\dual} } ,\end{align*} which equals what was obtained above. This exhibits the isomorphism as $\CC\dash$vector spaces, to see that this is in fact as isomorphism of $U(\lieg)\dash$modules we can use the fact that for $M\in \OO$, a twisted $\lieg\dash$action was defined as $$ \vector v\in M,~f\in M\dual,~ g\in \lieg \implies (g\cdot f)(\vector v) = f(\tau(g) \cdot \vector v) $$ for the transpose map $\tau$. This action can be "linearly extended" over direct products and tensor products by taking the action component-wise, and is thus preserved by all of the isomorphisms appearing above. Since the final terms $\bigoplus_{\lambda \in \lieh} \bigoplus _{\alpha + \beta = \lambda} M_\alpha\dual \tensor L_\beta\dual$ are identical, they carry the same action, and since they are preserved by the isomorphisms, working backwards shows that the actions on $(M\tensor L)\dual$ and $M\dual \tensor L\dual$ must also agree, yielding the desired isomorphism. $\qed$ # Humphreys 3.4 Show that $\Phi_{[\lambda]} \intersect \Phi^+$ is a positive system in the root system $\Phi_{[\lambda]}$, but the corresponding simple system $\Delta_{[\lambda]}$ may be unrelated to $\Delta$. > For a concrete example, take $\Phi$ of type $B_2$ with a short simple root $\alpha$ and a long simple root $\beta$. > If $\lambda \definedas \alpha/2$, check that $\Phi_{[\lambda]}$ contains just the four short roots in $\Phi$. ## Solution We would like to show the following two propositions: 1. $\Phi_{[\lambda]}^+ \definedas \Phi_{[\lambda]} \intersect \Phi^+$ is a positive system in $\Phi_{[\lambda]}$, 2. In general, the associated simple system $\Delta_{[\lambda]} \neq \Phi^+_{[\lambda]} \intersect \Delta$. ### Proof of Proposition 1 We'll use the definition that for an abstract root system $\Phi$, a positive system $\Phi^+$ is defined by picking a hyperplane $H$ not containing any roots and taking all roots on one side of this hyperplane. However, if every element of $\Phi^+$ is on one side of $H$, then any subset satisfies this property as well, thus $\Phi_{[\lambda]} \intersect \Phi^+$ consists only of positive roots and thus forms a positive system. ### Proof of Proposition 2 Concretely, we can realize $\Phi$ and $\Delta$ as subsets of $\RR^2$ in the following way: \begin{align*} \Phi &= P_1 \disjoint P_2 \definedas \theset{[1,0], [0, 1], [-1, 0], [0, -1]} \disjoint \theset{[1,1], [-1, 1], [1, -1], [-1, -1]} \\ \Delta &\definedas \theset{\alpha, \beta} \definedas \theset{[1, 0], [-1, 1]} ,\end{align*} where we note that $P_1$ consists of short roots (of norm 1) and $P_2$ of long roots (of norm $\sqrt{2}$) and we've chosen a simple system consisting of one short root and one long root. Now by definition, \begin{align*} \Phi_{[\lambda]} &\definedas \theset{\gamma \in \Phi \suchthat \inner{\lambda}{\gamma\dual} \in \ZZ}, \quad \quad \gamma\dual \definedas {2 \over \norm{\gamma}^2} ~\gamma, \\ \Delta_{[\lambda]} &\definedas \theset{\gamma \in \Delta \suchthat \inner{\lambda}{\gamma\dual} \in \ZZ} .\end{align*} Now choosing $\lambda \definedas {\alpha \over 2} = \thevector{\frac 1 2, 0}$, we now consider the inner products $\inner{\lambda}{\gamma\dual}$ for $\gamma \in \Phi$: Thus \begin{align*} \gamma_1 \in P_1 &\implies \inner{\thevector{\frac 1 2, 0}}{ 2 \gamma_1} = 2\qty{\frac 1 2} \inner{[1, 0]}{\gamma_1} = \qty{\gamma_1}_1 \in \theset{0, \pm 1} \in \ZZ \\ \gamma_2 \in P_2 &\implies \inner{\lambda}{\gamma_2\dual} = \inner{\thevector{\frac 1 2, 0}}{\frac 2 {\qty{\sqrt 2}^2}\thevector{\pm 1, \pm 1}} = \pm \frac 1 2 \not\in \ZZ \end{align*} where $(\gamma_1)_1$ denotes the first component of $\gamma_1$. We thus find that \begin{align*} \Phi_{[\lambda]} &= P_1 && \text{the short roots} \\ \Delta_{[\lambda]} = \Phi_{[\lambda]} \intersect \Delta &= \theset{\alpha} && \text{the single short simple root} .\end{align*} Choosing the following hyperplane $H$ not containing any root, we can choose a positive system: ![](figures/image_2020-04-26-16-35-07.png)\\ \begin{align*} \Phi^+ = \theset{\beta, \beta+ \alpha, \beta + 2\alpha, \alpha} \end{align*} where we can note that $\Phi^+ \intersect \Delta = \Delta$, since we've placed both simple roots on the positive side of this hyperplane by construction. But by taking roots on the positive side of this plane, we have \begin{align*} \Phi_{[\lambda]} = \theset{\alpha, -\alpha, \alpha+\beta, -\alpha-\beta} \implies \Phi^+_{[\lambda]} &= \theset{\alpha, \alpha+\beta} \end{align*} where we can now note that a simple system in *this* root system must still have rank 2, so we can take $\Delta_{[\lambda]} = \theset{\alpha, \alpha + \beta}$. But now we can note \begin{align*} \Delta_{[\lambda]} = \theset{\alpha, \alpha+ \beta} {\color{red}\neq } \theset{\alpha} = \theset{\alpha, \alpha+\beta} \intersect \theset{\alpha, \beta} =\Phi^+_{[\lambda]} \intersect \Delta ,\end{align*} which is what we wanted to show. $\qed$ # Humphreys 3.7 a. If a module $M$ has a standard filtration and there exists an epimorphism $\phi: M\to M(\lambda)$, prove that $\ker \phi$ admits a standard filtration. b. Show by example that when $\lieg = \liesl(2, \CC)$ that the existence of a monomorphism $\phi: M(\lambda) \to M$ where $M$ has a standard filtration fails to imply that $\coker \phi$ has a standard filtration. ## Solution > Haven't had a chance to work this out yet!